Which one of the following could be the animals included in the display, listed in the order of their pens, from firs...

Edunn on April 26, 2020

Game Set up

Can someone help me with the game set up? This was my first shot at a practice LSAT. My results were disappointing to say the least.

Replies
Create a free account to read and take part in forum discussions.

Already have an account? log in

Skylar on April 26, 2020

@Edunn, happy to help!

We often see great variance between students' scores from their first time taking an LSAT test to their final time after completing the lessons and practice, so don't get too worried just yet. The Logic Games section is especially designed in a way that allows practice and strategy to significantly improve one's score. I'll set up the game as you requested, and I'll also walk you through how I completed each question so that you can see various strategies.

First, let's break down the game setup:

Kittens: FGHJ
Puppies: RSTW
So, we have a total of 8 animals (4 kittens and 4 puppies), but we can only select 5 animals to put in the pens. This means we start out with any of the following possible combinations:
4 kittens and 1 puppy
3 kittens and 2 puppies
2 kittens and 3 puppies
1 kitten and 4 puppies

__ __ __ __ __ | __ __ __
1 2 3 4 5 OUT

Rule #1: Spots 1 and 5 = kittens (so F/G/H/J)
Rule #2: T cannot be next to any kitten (F/G/H/J). From Rule #1, we know that the 1st and 5th pens hold kittens. Therefore, T cannot be in pens 1 or 5. This also means that T cannot be in pens 2 or 4. So, if T is in, T must be 3rd and the 2nd and 4th spots must be puppies.
Rule #3: One of G/H is in, and the other is out.
Rule #4: W -> G(2). The contrapositive of this is: NOT G(2) -> NOT W.

__ __ __ __ __ | (G/H) __ __
1 2 3 4 5 OUT
K K

Now, let's revisit the possible combinations we wrote out before we looked at our rules. We can eliminate the "1 kitten and 4 puppies" option because Rule #1 tells us that we always have at least 2 kittens in. We can also eliminate the "4 kittens and 1 puppy" option because Rule #3 tells us that we cannot have all 4 puppies in. This leaves our options as:
3 kittens and 2 puppies
2 kittens and 3 puppies

Based on this game setup, let's look at how to approach the questions efficiently:

QUESTION 1:
With these types of questions, we should go through rule-by-rule and eliminate any answer choices that break the rules.
- (E) breaks Rule #1 because it places T (a puppy) in the 1st spot.
- (D) breaks Rule #2 because it places T next to G (a kitten).
- (B) breaks Rule #3 because it places both G and H in.
- (C) breaks Rule #4 because W is in, but G is not in the 2nd spot.
- This leaves (A) as the correct answer. We should also write this diagram to the side quickly to use as reference for later questions if needed.

QUESTION 2:
This question places Wags in, so let's start at Rule #4, where we have a Sufficient condition of "W."
- Rule #4 tells us: W -> G(2). So, we know that G is in and placed in the 2nd spot.
- Rule #3 tells us that if G is in, H cannot be in. So H is out.
- Rule #2 tells us that if T is in, it would be placed 3rd and the 2nd and 4th spots would have to be dogs. However, we established that G (a kitten) is in the 2nd spot in this scenario, so T must be out.
- Since we can conclude that if W is in, both H and T must be out, (B) is correct.

QUESTION 3:
This refers to the second possible combination we identified - 2 kittens and 3 puppies.
- We know from Rule #1 that the 2 kittens must be in spots 1 and 5.
- Therefore, the 3 puppies fill spots 2, 3, and 4.
- So, we can first eliminate (A) because it places a kitten in spot 2.
- We know from Question 2 that if W is in, then G will be second, which again places a kitten in spot 2. So, (E) can be eliminated and W must be out.
- This leaves R, S, and T as the 3 puppies that must be in.
- Rule #2 tells us that if T is in, it must be placed 3rd. So, (D) is correct.

QUESTION 4:
This places F next to J.
- We know that we can only have a maximum of 3 kittens and 2 of these are assigned to spots 1 and 5. So, F and J must either be in spots 1 and 2 or in spots 4 and 5.
- Since a kitten must be in a spot 2-4, T cannot be in.
- W also cannot be in, because it would require that G is placed 2nd. This is impossible because if F and J are in spots 1 and 2, spot 2 would already be occupied. If F and J are in spots 4 and 5, there would only be one spot left open for a kitten and it would have to be spot 1. So, G could not go 2nd.
- Therefore, (E) Wags is correct.

QUESTION 5:
This tells us that J is out.
- We know that either G or H is always out. In addition to J now being out, this means that only 2 kittens are in. Following Rule #1, we know they are in the 1st and 5th spots. So (A) is out.
- As in our last question, this means that W must also be out because it would require G (a kitten) to be placed 2nd. So (E) is out.
- This leaves R, S, and T to be the three puppies in. Again, Rule #3 says that if T is in, T must be third. This eliminates (C) and (D).
- Therefore, (B) is the correct answer.

QUESTION 6:
This tells us that S is out.
- Let's see what happens if we try to put the remaining 3 puppies in. We notice that we run into a problem trying to place both T and W in. W requires that G be placed 2nd, which means that T cannot go 3rd.
- However, this same issue would happen if we tried to put the remaining 3 kittens in instead, because T cannot be in unless there are a total of three puppies in. So, T has to be out.
- This leaves (F/J) to go 1st and 5th, G to go 2nd, and (W/R) to go 3rd and 4th.
- So, (C) is correct.

Does that make sense? Hope this helps with the rest of your studies! Please reach out with any additional questions.

Henry on September 26 at 12:26AM

Thank you for taking the time to explain how you got the correct answers. While the videos are helpful, they can be frustrating at times when they rush through and omit key parts to get to the correct answer. I prefer your approach better.